Compilation 509+ MCAT Study Habits

This forum made possible through the generous support of SDN members, donors, and sponsors. Thank you.

supremus

Full Member
7+ Year Member
Joined
Jan 13, 2015
Messages
86
Reaction score
77
Use the template below:

1) Your individual scores and composite score
2) The study method used for each section
3) What materials you used for each section(Kaplan, TPR, Examkrackers, AAMC, TBR, etc)
4) Which practice tests did you use? (Optional: include scores)
5) What was your undergraduate major?
6) Any other tips you may have for those of us who still have this test lurking over us?
7) How long did you study for the MCAT?

Members don't see this ad.
 
Last edited:
  • Like
Reactions: 28 users
Solution
Use the template below:

1) Scores:
  • C/P: 128
  • CARS: 131
  • Bio: 132
  • Psych: 127
  • Total: 518
2) The study method used for each section
  • Overall: I read every page of the review books and took notes for chem and psych, as they were my weakest sections. For any section that I had never gone over in school, I read and took notes (such as the metabolism sections). I would go back and forth with the Kaplan online videos/lessons, their practice problems, and the book in order to really memorize the info
  • Chem/Phys: this was easily my weakest section to begin with, so I went back to the basics (for chem)
  • CARS: the Kaplan tips on how to work with the CARS section were invaluable. Their hour-long MCAT channel...
1) Your individual scores and composite score
CP: 131 CARS: 127 BS: 130 PS: 130; Overall: 518 (97th percentile)

2) The study method used for each section
In the beginning of the summer (around mid-May), I began a very thorough review of all of the material. Then, I zoned in on specific topics that I knew I was fuzzy/unfamiliar with, and I studied those further.
Afterwards, I started doing a ton of passages (mostly from the Berkeley Review and EK). I took TBR prep class, and we were assigned homework (passages given to us in class and within the review books), which I compiled into a schedule and followed throughout the summer. As for specifics...

3) What materials you used for each section
I used TBR, TPR, and EK books.
TPR: This was what I used for my general, comprehensive overview. It's extremely dense and has lots of extraneous information, but it covers pretty much everything you need to know.
TBR: Where the Berkeley Review seriously shines is the sheer amount of practice problems that they give you. For each topic within each subject, there are roughly 15 passages and 100 total practice questions. Many of them are tricky, many will make you facepalm because you thought you had the right answer, and all of them are helpful. The Berkeley Review helped me TONS for discrete questions and learning to navigate passages. The questions will make you think very critically, and they're significantly more difficult than the problems you'll see on the real thing on test day. It really prepares you for those WTF questions that you might not expect, and you'll have a reliable test bank of knowledge ready to be used. Admittedly, I hardly read the books for content (extremely dense and overly detailed), but I imagine that it could really help with reinforcing concepts. One of my Berkeley Review teachers openly admitted that she (almost) never read the books, her entire study schedule was based on practice, practice, practice.
EK: I read all of these books to reinforce what I already knew. Furthermore, the 30 minute practice passages are EXTREMELY, EXTREMELY helpful. The questions are unreasonably difficult (much more difficult than AAMC-level questions), but they prepare you for the few difficult questions you might encounter on test day. The star of these practice passages are the passages themselves. After taking the AAMC FL and the real thing, EK did (in my opinion) the best job of capturing the essence of what the new MCAT passages are all about. Many of their passages are experiment/biochem heavy, and reading through them and getting used to them really helped me tackle those types of passages on the real deal.

C/P: Typically one of my weaker sections because I suck at physics (sound, doppler, light, optics, I hate it all). But, very thankfully, there was a strong biochem and gen chem emphasis on my exam (two of my strongest subjects). My preparation for physics mostly consisted of reviewing using TPR and EK, and using the practice problems from TBR and thoroughly trying to understand the answers. Since biochem and gen chem are my stronger suits, I didn't review much, mainly used practice problems as my way of studying those subjects.
CARS: Also one of my weaker sections, as reflected by my score. I did really well on CARS for some of my practice FLs, and really poorly on others. I can't say much for CARS besides practicing helps a lot, and really grasping the main idea of each passage is the key to doing well.
BS/B: Pure memorization when it came to review for bio. No other way around it, really. As for ochem and biochem, my courses in each subject really solidified my understanding of both, so they weren't so much of a problem for me. Again, I'd like to emphasize that doing practice problems will ultimately determine how well you know the material, being able to regurgitate information just doesn't cut it. Furthermore, Khan Academy videos does an amazing job of organizing all of the biochem topics you need to know. If you're rusty on gluconeogenesis, glycogenolysis, fatty acid oxidation, etc etc, then use their videos. Know enzymes. You never know what kind of discrete questions you could get. *hint hint*
P/S: Honestly, I'm not sure what happened here. I put off studying for P/S very last minute, I spent 3-4 days before my exam memorizing all of the theories and terms I had studied/reviewed from TPR. Furthermore, I went to the Khan Academy video website and watched pretty much every single video on topics that I was uncomfortable or unfamiliar with. I also did all of the P/S Khan Academy passages. Somehow, I pulled that 130 out of my butt. Be prepared for more sociology than you would expect/be familiar with. Also, I treated some of the passages as almost a CARS/P/S combination, and that worked for me. It seemed as though the passage writers had opinions which could be used to answer the questions.. Not sure if anyone else felt the same way. Also, I have heard that P/S has a very generous curve, but I'm not sure how accurate that assertion is.

4) Which practice tests did you use?

TPR (3 free ones with review books): 502, 503, 503
EK (FL 1, 2, 3): 82%, 71%, 73%
TBR: 509, 511, 513, 515
AAMC Official Guide: 26, 26, 22, 22
AAMC FL: 86%, 83%, 85%, 80%

5) What was your undergraduate major?
Neuroscience

6) Any other tips you may have for those of us who still have this test lurking over us?
**Disclaimer: I am a terrible standardized test taker. In the past, I've lacked the discipline to study in the long term, and I'm really good at cramming for exams and waiting until the last minute. All I can say is that if you're anything like me, suck it up and put in the effort. It will go a long way.**

Cannot emphasize enough, PRACTICE > CONTENT REVIEW. No matter how much you feel like you've mastered the material, it doesn't matter if you can't apply it to AAMC-like question types. This is why practice is miles more important if you have the general concepts down.

Get into a habit of waking up as early as you would on test day and doing practice problems/ tests to mirror testing conditions. This will help you immensely and eliminate some of the nerves that might be associated with test day.

Eat healthy and don't drink too much caffeine (less than you would on a normal day) on test day. The adrenaline is plenty sufficient to keep you awake and wired. I drank too much coffee and was shaking for half of the test.

Did I mention you should do practice problems? Because you should.

7) 3 months, from mid-May to mid-August. 2 hours of classes per week for 5-6 days per week, and 6-7 hours of studying each day on top of that.
How representative are the TBR full length practice test to the real test?
 
1) SCORE: 517 (96th percentile)
131/128/128/130 (April 2016)

2) Method:
a) I started off by doing content review with the Kaplan prep books. Read everything in pretty good detail and wrote down abbreviated notes for future study.
b) Then, after doing the end of chapter questions, I focused on the things I was struggling with conceptually - watching Khan Academy videos really helped here. I actually got into the habit of watching Khan Academy videos while I ate or cleaned my apartment so that at least some of it would soak in.
c) When I felt someone comfortable with the content, I started doing scored practice exams for both Kaplan and TPR. I didn't do too well on them at first, and found that I was forgetting a lot of stuff that I previously remember or understood. So, I turned to the MCAT quicksheets that Kaplan provided and I was sure to memorize every single page of that/ understand all of it. This helped a lot because it gave me a really solid foundation of knowledge that was easy to build on.
d) After that, I reviewed using exclusively the AAMC book with the various topic headings. I went over them again and again making sure that I was able to at least recognize every single topic that they mentioned. If I didn't understand something Khan Academy was usually my go to.
e) Lastly, I took the AAMC resources (practice test, sample exam, section banks). Since I was scoring well on those I just went and reviewed everything in the final week before my exam.

3) Materials:
I think Kaplan was better for life sciences, but Princeton for CARS/ Psych.
AAMC Section Bank was my favorite resource

4) TPR/ Kaplan/ AAMC
Most of my TPR/ Kaplan were around 500, but showed steady rates of improvement (which is really all you should be looking for with those)
AAMC Sample : 83%, 92%, 92%, 86%
AAMC Practice: 130, 128, 130, 127

5) Major
Biochemistry and Sociology (not for the MCAT, but it definitely helped to have both of these)

6) Don't get dragged down too much by the TPR and Kaplan resources. They make it a lot harder than the real thing and that can sometimes make you less motivated to study. Keep at it - you're probably doing a lot better than you think. My favorite resource were the section banks, I think they are harder than the real thing but in a way that introduces you to important question types/ notation that will be essential for you to know on test day.

7) I'd say about 3 months. First month was all content review. Second month was Quicksheets/ practice exams/ review. Last month was all about the AAMC materials.

If you have any questions for me let me know!
 
  • Like
Reactions: 4 users
Congrats on Score @GonFreecs - did you ever see the AAMC flashcards? Wondering how relevant they are (I bought them; have them with me everywhere - have the "got correct" pile and the "need to understand" pile... at least I have 2 piles already!)
 
Members don't see this ad :)
Congrats on Score @GonFreecs - did you ever see the AAMC flashcards? Wondering how relevant they are (I bought them; have them with me everywhere - have the "got correct" pile and the "need to understand" pile... at least I have 2 piles already!)

Thanks so much! I actually have not looked at the flash cards but from what I understand there are only 150 cards. If so, I think its a great idea to use them! Like I said earlier, I think building a strong foundation of knowledge is much more important than knowing a bunch of random stuff!
 
The cards have graphs, tables and stuff along with all that psych/soc :/ lol - thanks again
 
1 April 2016--test date.

1) Overall Score: 524 (This is listed as the 100th percentile on my score report but that feels weird to say): (132, 130, 131, 131)

2) Did relatively little content review. Practice practice practice. FOR THE LOVE OF GOD learn all the content in the section banks. I thought "if it's in the section bank there's no way they'd put that same exact content on the real test" but I was freaking wrong. It was there.

3) Kaplan and AAMC Materials


4) (On all of these, C/P and B/B were consistently one point higher than CARS and P/S. Just random variation btw FL3-6)
Kap Diagnostic (before taking ochem2 or biochem, no studying): 505
Kap FL2: 509
Kap FL3: 512
Kap FL4: 512
Kap FL5: 512
Kap FL6: 512 (this drove me absolutely insane.)
Official Guide Half-Length Exam: 516 (Used percentage conversion table)
AAMC Scored: 522
AAMC Unscored: 521 (Used percentage conversion table. I don't have access to my percentages anymore but they were all in the mid-90's except P/S which was mid-80's)
Actual: 524 (132, 130, 131, 131) *Fist pump*

5) Statistics

6) Do everything AAMC you can find. I didn't go all the way through it and walking out of the test I was kicking myself for it hard.
I did not feel good about the test because there were quite a few that I missed that would have been simple if I had just gone through the section banks I would have gotten more of the straight up content. If you have to pick one section to focus on for content, make it P/S. P/S was absolutely the most content-specific of all of these. I had very little OChem on mine and what was there was super basic. My biggest advice is to ignore this whole "MCAT needs to be your whole life 4-8 hours a day for 2 months" because that is crap. Practice, study, and put in the time that is necessary, but if you drive yourself insane with it, you will only be hurting yourself. The most important thing, as far as I can tell, is to stay sane. Don't panic. Do as much timed practice as possible because the moments that separate the 500-510 and the 520+ are ones where you are sitting there in front of the test watching the clock count down and have no freaking clue what is going on in that passage. Invent a shorthand, a method, something so that you can handle that moment. Learning how to stay calm in that moment is more important than any content you could memorize.

Don't lose your soul to this test.

7) 2 months or so.

Oh and I felt like crap leaving the testing center. I actually debated cancelling my score.
 
Last edited:
  • Like
Reactions: 21 users
1) Score
518
132/127/131/128
Old MCAT in 2013 14/8/14 (PS/Verbal/BS)
2) Method
Traditional content review + practice problems. Will elaborate on the materials section
3) Materials
EK General Chemistry, Physics, Biology // Kaplan Ochem, Biochem // TPR Psych and Soc // Khan Academy videos and problems // TPR and EK Verbal
Honestly, I think TPR Psych/Soc does not cover enough material for psych and sociology. I had 3 questions on real MCAT where I didn't know what the term being referred to was. I found Khan Academy videos very helpful when filling in the gap between the TPR and AAMC expectations.
I didn't struggle much with physics, gchem, ochem, old MCAT bio materials because of previous knowledge. I did only practice problems from EK books
Biochem was new to me. I went over them in EK Bio as well as Kaplan Biochem. I honestly thought that biochem discussed in EK bio was somewhat limited. Kaplan Biochem content was more in-depth, but I felt that it's better to be safe than sorry.
I also used Khan Academy practice passages. I found that psych/socio passages were pretty helpful. Bio passages were alright. Chem/Physics passages were either too hard or too easy.
I always struggled with CARS as I'm an ESL. I finished TPRH verbal and did half of EK verbal
4) Practice tests
AAMC Scored 131/126/129/129
AAMC Unscored 95%/81%/93%/86%
TPR 1 128/125/129/129 - Of note, I felt TPR practice test was very different from AAMC MCAT. Verbal was too hard. Psych/Soc focused too much on memorizing names of theorists.
5) Major
Physiology.
6) Tips
First of all. Know the contents in and out. Then practice applying the knowledge with problems. Always challenge yourself with difficult questions. Do not skip reviewing process because you think a problem is too hard/unfair.
For CARS -- As you can see from my previous old MCAT verbal score, I am not good with verbal, but I found highlighting very helpful. 2 weeks before the test, I tried using highlighting function when reading passages. The passages came to me much easier and I was able to retain the information for longer.
Real MCAT will make you feel miserable. But do not worry. For both of my MCAT experience, I felt miserable for some sections (PS and BS from old MCAT, CARS and Psych/Soc from new MCAT). I completely guessed on 1 passage on CARS and even thought about voiding the exam. But don't give up. The scales will actually work in your favor, and your scores will be higher than you thought.
7) Length of Studying
3.5 months. Worked 50 hours a week for the first 2.5 months. Worked part time for the rest.
I think I could pull it with lots of stuff on my plate due to retained knowledge from my major and old MCAT studying process.
 
  • Like
Reactions: 1 user
1) SCORE:
516: 127/130/129/130

2) STUDY METHOD:
CARS - ignored.
Chem/Phys/Bio - content review and practice problems
Social section - content review, creation of detailed notes, creation of abridged notes, practice problems.

3) What materials you used for each section(Kaplan, TPR, Examkrackers, AAMC, TBR, etc)
CARS - none
M Prep (mcatquestion.com) for lectures and primary content review of sciences and social studies. M Prep Question bank used for physics and chemistry practice problems. Read through Exam Krackers prep book when M Prep did not give enough detail on certain topics. Used EK end of chapter quizzes to gauge comfort. Had old kaplan books from a friend (for old exam) if I needed more detail, but rarely did.

Used quizlet to create flashcards/use other flashcards to memorize amino acids.

Used practice problems to find lapses in high yield areas, studied them, did the same problems then similar problems then moved on. Used primarily AAMC section/content packs for practice.

I did about 2.5 months study time. 6 Weeks content review with M prep practice problems throughout. 4 weeks practice only (AAMC practice problems) to determine high yield learning as above.

For social sciences I made detailed notes in OneNote, then revised them since they can become just a list of definitions in the M Prep course - tying concepts together so remembering one thing leads to knowing multiple similar things.

4) Which practice tests did you use? (Optional: include scores)
I took 3 practice tests if you count the diagnostic.

Kaplan Diagnostic before studying 498 (half length)
Kaplan Full Length after content review 504 (4 weeks before test date)
AAMC Practice Test #1, one week before test date - 512 (128/130/128/126)

After AAMC practice test, with 1 week to test date, I focused only on the social section to improve this. Memorized theories, names, and grouped concepts mentally to recall in chunks. Sociology, more than many sections, is a "know it or you don't" section. If you don't know the content you simply can't answer the questions much more often. Didn't really do much practice problems, did flashcard-like review to ensure improvement.

5) What was your undergraduate major?
Biology, BS. Graduated spring 2013

6) Any other tips you may have for those of us who still have this test lurking over us?
Don't practice studying for a test, practice taking a test.

7) How long did you study for the MCAT?
10 weeks
 
Last edited:
  • Like
Reactions: 1 user
1) Scores
509, 128/129/126/126

2) Study method
CARS: Did practice passages to make sure I felt comfortable, but did not "study."
Worked my way through the Examkrackers books chapter by chapter for the rest of the material, took all 30 min chapter exams and did all in book practice problems.
After week 5, took an EK full length every weekend. Took the AAMC full length the week before my exam. When I started to burn out 2 weeks before the exam date, I supplemented with Khan Academy videos for my weakest subjects.

3) What materials you used for each section
Examkrackers 9th edition
AAMC Scored full length, section banks
Khan Academy videos
McGraw Hill 500 MCAT questions in General Chemistry (had been a long time since I took Gen Chem, wanted extra question practice)

4) Which practice tests did you use? (Optional: include scores)

EK 1-3
1- 59%
2- 70%
3- 69%

AAMC FL Scored- 506

5) What was your undergraduate major?
Business Administration

6) Any other tips you may have for those of us who still have this test lurking over us?
If I could have done it over again, I would have completed content review earlier. I didn't have a strict content review period- instead, I worked my way through the books up until 2 weeks before my exam. I wish I had more time dedicated specifically to practice questions only. Also, I started the AAMC materials too late (10 days before the test). I blame being in school/writing my thesis this semester, but I can't help but feel like I had the ability to score higher if I had devoted more time to doing practice questions.

That being said, I think that doing timed practice questions/full lengths during content review has some merit. I am so glad that I didn't wait until the very end to start taking them. Spacing them throughout your studying gives you a good feel for time management/areas of weakness.

7) How long did you study for the MCAT?
10 weeks
 
  • Like
Reactions: 1 user
1) Score: 523 (131, 131, 132, 129)

2/3) Method: Examkrackers for Physics, Chem and Biology. TPR for Psychology, and Princeton Review Hyperlearning book for CARS. I basically went through every chapter in Examkrackers twice, and took notes/made Anki flashcards for all of them (which I tried to study weekly). I reread the TPR Psych book 2-3 times, and tried to do a few CARS passages every week (although CARS was always my strength so I didn't practice too much). I used Khan Academy when needed. If I could re-do I would definitely spend less time on content (which everyone says and yet I didn't listen to!) I feel like after your first review of everything, you should really just PRACTICE and focus in on things you don't understand from your practice tests/questions. I did that the last 1.5 months and felt like I got more out of it than the previous 6+ months of content review combined.

4) Practice tests: EK1-4 (around 85% on all of them), Berkeley Review 1 (would not recommend AT ALL, don't even remember my score), AAMC Sample Test (90%), AAMC Practice Exam (522), around 75-80% on the Section Banks, around 90% on the question packs.

5) Major: Molecular Biology

6) Tips: PRACTICE. I know it feels like you need to go through all of the content first, but if you finished your pre-reqs in the recent past, then I promise you, you WILL get more out just doing a ton of practice exams and questions. Save the AAMC ones for the last ~2 months, but before that, take advantage of any other company's practice exams and question banks you can find. And the key is to REVIEW your exams front and back. Make a flashcards/note for every question you missed and study that topic. Your yield will be much higher than simply going through content chapter by chapter in these review books. But the key is REVIEW. You have to keep rereading your notes and flashcards every week or so to make sure it sticks!

7) How long did you study for the MCAT? 7 months, by total accident. I studied only 1-2 hours a day, maybe 4 hours a day on weekends (unless it was a practice exam day of course). I had set aside 5 months to study (from September to January) knowing that I would be doing short bursts (I'm not a student, but I have 2 jobs and a ton of other commitments). However, the storm canceled my test center in January and I had booked a vacation over the makeup date, so I ended up studying an extra 2 months by accident.
 
  • Like
Reactions: 6 users
Members don't see this ad :)
was it helpful to section off the material weekly? Or daily? or hourly? I know I tend to learn material better when I focus on it completely, so if I devoted a month to Biochem (just an example) I'd know biochem cold. But I'm worried about attrition over time come test day.

What do you think?
 
1 April 2016--test date.

1) Overall Score: 524 (This is listed as the 100th percentile on my score report but that feels weird to say): (132, 130, 131, 131)

2) Did relatively little content review. Practice practice practice. FOR THE LOVE OF GOD learn all the content in the section banks. I thought "if it's in the section bank there's no way they'd put that same exact content on the real test" but I was freaking wrong. It was there.

3) Kaplan and AAMC Materials


4) (On all of these, C/P and B/B were consistently one point higher than CARS and P/S. Just random variation btw FL3-6)
Kap Diagnostic (before taking ochem2 or biochem, no studying): 505
Kap FL2: 509
Kap FL3: 512
Kap FL4: 512
Kap FL5: 512
Kap FL6: 512 (this drove me absolutely insane.)
Official Guide Half-Length Exam: 516 (Used percentage conversion table)
AAMC Scored: 522
AAMC Unscored: 521 (Used percentage conversion table. I don't have access to my percentages anymore but they were all in the mid-90's except P/S which was mid-80's)
Actual: 524 (132, 130, 131, 131) *Fist pump*

5) Statistics

6) Do everything AAMC you can find. I didn't go all the way through it and walking out of the test I was kicking myself for it hard.
I did not feel good about the test because there were quite a few that I missed that would have been simple if I had just gone through the section banks I would have gotten more of the straight up content. If you have to pick one section to focus on for content, make it P/S. P/S was absolutely the most content-specific of all of these. I had very little OChem on mine and what was there was super basic. My biggest advice is to ignore this whole "MCAT needs to be your whole life 4-8 hours a day for 2 months" because that is crap. Practice, study, and put in the time that is necessary, but if you drive yourself insane with it, you will only be hurting yourself. The most important thing, as far as I can tell, is to stay sane. Don't panic. Do as much timed practice as possible because the moments that separate the 500-510 and the 520+ are ones where you are sitting there in front of the test watching the clock count down and have no freaking clue what is going on in that passage. Invent a shorthand, a method, something so that you can handle that moment. Learning how to stay calm in that moment is more important than any content you could memorize.

Don't lose your soul to this test.

7) 2 months or so.

Oh and I felt like crap leaving the testing center. I actually debated cancelling my score.

Congrats on that stellar score! My test date it June 18 (3-4 weeks out) and I'm trying to AT LEAST bring my practice test score up from the 506-508 range to 510+ if possible. I just got the section banks, and I was curious as to how you utilized them? Besides just answering the questions.

My plan was to go through the section banks, and just read up on anything I didn't understand (I have the EK book set, love it so far.) I plan on taking two practice tests a week as well (I still haven't taken EK 2-4 FLs and the AAMC unscored FL) and then taking the scored AAMC FL again the week of my exam to (in theory) boost my confidence.
 
Hi there. Long-time lurker here; this is my first post. I’ve gotten so much great information off of this site, I figured I was long past due to give back.

[I should note here that I’m much older than your average pre-med — 39 — and I think this conveyed both advantages and disadvantages, which I’ll try to touch on at the appropriate time.]



1) Your individual scores and composite score

PS: 127
CARS: 132
BS: 131
Psych: 132

Total: 522 (99th percentile)



2) The study method used for each section

All sections
: I did every practice test I could get my hands on, especially the stuff released by the AAMC.

As well, a few friends and I did a course here in LA called Swartwood. I do NOT recommend this course. They are incredibly disorganized, paranoid, and outright bizarre. None of us — not one of us — was glad we took this course. (I do think that their method makes sense for CARS, though. I’ll explain below.)


PS: This has always been my Achilles heel (due to various reasons including the fact that I have some weird mental problem with basic arithmetic… I’m quite capable of adding 2 + 3 and getting 6). So, I pushed myself harder in this section than in any other. Luckily, it was more conceptual and less math-y than I expected, though you do have to have those equations memorized, because you never know which one will come up!

I made flash cards for the equations. The physics stack was huge, but I just made myself drill it (alone as well as with friends). The gen chem stack was not so big. I also glanced over solubility rules and such just to make sure I remembered them, but didn’t memorize them flat-out.

Other than that: practice practice practice. I went back over anything that felt rusty from my classes, and really tried to avoid plug-and-chug. What do I mean by this? I mean that, tempting as it was at times, I never allowed myself to just throw in an equation and get a result without knowing why that was the result. This is very very important for the MCAT style of reasoning.

Also (and this applies to both PS and BS), I memorized the **** out of the amino acids. I used a couple of ipad apps — both entitled “Amino Acids," helpfully — and tried to be able to recognize them whether they were drawn in bond-line, 3D, “old-school” Lewis-ish notation, etc. As well, there are some unique features to certain amino acids, such as glycine being achiral and ambivalent, or proline causing kinks in peptide chains. Know those. And know the single-letter codes! I messed up on that.


CARS: This is one of the places where I think my age came in handy. I’ve been a lifelong reader, so I had a sort of store of “crystallized knowledge” upon which to draw. Other reading-oriented friends (though younger) had a similarly good time with CARS. I’ll admit to not really studying for this section, other than by doing the practice tests.

A passing familiarity with a wide variety of subjects is a boon, but it’s admittedly difficult to learn “a little about a lot” without having time. With that said, however, there is a method (gleaned from Swartwood) that can help those among you who don’t have time for a lot of casual reading (and as pre-meds, that’s probably most of you): Keep it stupid!

I took notes that were really, really dumb. Like, first paragraph: “guy likes Rembrant.” Second paragraph: “Other guy thinks first guy sux.” Literally, I wrote stuff like that. Just try to boil each paragraph down to its essential characteristic, in the most childish, unremarkable terms possible.
Also, If you find a metaphor used over and over again in a passage, hone in on that. It’s probably important.

For some context, I have a super math-oriented friend who hates reading, and using the stupidity method, he was able to boost his scores considerably over time. He just had to stop trying so hard.


BS: See above regarding the amino acids. Applies to both sections. Other than that, I refamiliarized myself with cellular stuff, DNA/RNA, basic physiology. The kidney can be kind of complex, make sure you understand what's going on there. Also, know what actions take place in which part of the digestive system. I was a psych major, so I already had a passing familiarity with neuro stuff, but they definitely seemed to like this topic as well.

But definitely, definitely study a lot of biochem. They went hard on it. Know the essentials: which steps of glycolysis are irreversible, for example. Anything unusual or that ”sticks out,” try to memorize.

We made flash cards for the hormones, cell types, and steps in spermatogenesis and oogenesis.


Psych: I was a psych major (having completed my major only two years ago), so I was kind of ”psyched” on this section. :naughty:
I did the practice tests and made sure to memorize key terms (as laid out in the ExamKrackers books).



3) What materials you used for each section(Kaplan, TPR, Examkrackers, AAMC, TBR, etc)

As mentioned above, we did every (relevant) AAMC test. They put out bundles collected from older tests; we did those. The 120 questions; we did those. The “actual” practice tests; definitely did those. Some parts we did twice, just in case.

Otherwise: The ExamKrackers books were great, though I feel they were a bit lax in the biochem department. Also, they seemed to be guessing a bit in psych, but I think that was the case with everybody. As time goes on and more tests are released, prep companies will get a better feel for what that section looks like.

We also had some tests from Swartwood, which were… okay, I guess. They were riddled with typos and often had wrong answers. Also, they couldn’t get their act together in time to give us computer-based tests, so we did them on paper. And out of order. With long-ass breaks. And sometimes not all the same sections on the same day. Kind of wack, but it got us thinking about the material, so it was still sorta useful in that sense.

Khan academy has some great info and good practice tests, but they’re way too broad in scope. They need to narrow it down a bit (which I’m sure will happen with time). But I found their stuff useful for finding out more about specific topics about which I was confused or felt under-prepared. In other words, come to Khan somewhat primed and be targeted, otherwise you could get lost in the mountain of information.

Princeton has one free test, and we did that. It was horrible. Horrible! Demoralizing. Awful. And 90 times harder than the real MCAT. Like, it was crazy.

One thing that was useful was that we combined material to simulate test conditions. So, we’d take 52-ish questions from an old AAMC Physics section, same from old Verbal and Bio sections, and combine them with ExamKrackers psych to make a “whole” test. It was a way to not only study material, but to get the timing down. More about this later.



4) Which practice tests did you use? (Optional: include scores)

Oops. See above.

I don’t remember my scores. I feel like they were decent, but lower than on test day overall, with the possible exception of the 120 questions. Those were hard.



5) What was your undergraduate major?

Psych, with a few extra neuro courses thrown in for good measure.



6) Any other tips you may have for those of us who still have this test lurking over us?

Yes:

I think that some of the prep companies may be overestimating the complexity of the questions on the MCAT. They’ve all been racing to catch up to the new format and stylistic changes (some small, some big), and it’s a bit chaotic out there. But one thing I noticed was that most of their material was harder than the actual MCAT. So, don’t be discouraged if your practice scores are less than you want/expect. Stay strong and keep working! You CAN do this, it’s doable. And you may just be surprised by the outcome.

My exam had a heavy biochem emphasis in both the PS and BS sections. Study that ****. If you haven’t taken a course in it, you;ll be at a disadvantage, but can probably still learn the essentials.

Definitely memorize the amino acids!

I definitely think that some random “experimental” problems turned up. If it’s not covered in the practice materials (or the AAMC syllabus, which I barely touched), try quickly and move on. For example, if something were to show up about (hypothetically) about diffraction gratings where they don’t give you any equations (hypothetically), I wouldn’t sweat it.

Also, remember: On test day, everyone is just as nervous as you are! So even though the test is not “curved” as such, your percentile may end up being higher than you think.

I found that the psych threw a lot of people off. I think as more universities acclimate to this new MCAT, they’ll put more premed-oriented psych classes in place. In the meantime, focus on the terms, and the rest is critical thinking. And a lot of common sense.

(Actually, that may be a good thing to focus on: Figure out what parts of psych diverge from common sense, and make sure you remember those.)

PS: I never took a sociology course, nor did I take social psych. Ain’t necessary.

Watch out for fatigue during CARS. Have a small snack beforehand, even if you’re not hungry. Pee, even if you don’t think you have to.

And speaking of that stuff, have a good breakfast, lots of protein. I had eggs and a piece of fruit. Little bit of caffeine. (I had too much caffeine during lunch and was over-jacked on the bio section, so be careful)…

This is important: If your test is at 8am, get up early way in advance! I wanted to make sure I had zero “fuzz” on the brain at 8am, so I practiced getting up earlier and earlier as time went on. By the week before the test, I was waking up at 5:30 every day. it sucked but I think it helped, because I was well into mid-morning by the time 8am came around, and had had time to eat properly, shower, and feel generally as fresh as possible.

GET THERE EARLY. Rushing will throw you way off, emotion- and stress-wise!

Also, and this is very important (IMHO)… review your tests with your friends/study buddies! We went over everything that any of us (3 or 4 of us) got wrong, and it was immensely helpful. Guaranteed, someone in your group will see something differently, have some insight, or otherwise be able to point out something you missed. And it will stick.

One last thing: The highlighting was slightly different on the real test. You have to click an extra time to highlight and de-highlight things, for some reason. Not a game-changer, but slightly annoying.



7) How long did you study for the MCAT?

Prep course Feb-May (took it May 22nd), ramped up big time towards the last week. Then barely studied on the second-to-last day before the test, other than to go over some equations. Day before, went to the movies, relaxed.
Hey! I was actually wondering about Swartwood. I can't find a lot on them, but a lot of my friends have said positive things about them. Did you take the course only taught by John? Swartwood only or whatever? I'm in between that and Princeton Review.

Also, do you think doing an in person class is worth it? You seem to have had the self study/in person experience.

Thanks so much!
 
Congrats on that stellar score! My test date it June 18 (3-4 weeks out) and I'm trying to AT LEAST bring my practice test score up from the 506-508 range to 510+ if possible. I just got the section banks, and I was curious as to how you utilized them? Besides just answering the questions.

My plan was to go through the section banks, and just read up on anything I didn't understand (I have the EK book set, love it so far.) I plan on taking two practice tests a week as well (I still haven't taken EK 2-4 FLs and the AAMC unscored FL) and then taking the scored AAMC FL again the week of my exam to (in theory) boost my confidence.

Thanks! My problem was that I didn't use the section banks and missed at least four content-based questions (I confirmed the answers online and know for sure that I missed them) that I would have gotten no problem if I had at least scanned all the way through the section banks (Stuff like knowing what K_d is). Use them however you learn best, but the most important thing is to make sure you are at least familiar with the content inside. If you have lots of time to spend, use them as thought-process trainers. If you don't have too much time, just make sure to at least scan and learn the content they contain. Either way, make your way all the way through them; it would've helped me on chem and bio. It's weird, I KNOW that I missed two straight-up content q's on C/P but still got the 132. Take comfort in that, I suppose.
 
  • Like
Reactions: 1 user
SCORE: 516 (127/131/128/130), taken April 23rd

STUDY METHOD: I mainly studied by myself. I work full time, so it was a lot of time management making sure that I made enough hours in the day to study. A typical day was: wake up at 4/5A, study for a couple hours, do some yoga, go to work at 8:00ish, work until 1:15ish, study during my lunch break, work again until all my patients left (I work in a clinic), and then go home and study more. I took some days off here and there just to avoid burnout (sometimes you need to take a break to watch a whole season of Daria). I also studied with a girl I worked with on weekends/my day off during the week when we didn't have meetings/other stuff to do. That was mainly us getting together and going over stuff that we'd missed on the practice test the weekend before or topics we were both unclear on.

I also tried to integrate as much of the material into my life as possible. I'd never taken sociology and my last chemistry class was in 2010, so merely bringing up the material kind of refreshed it. I taught my husband all the psych/soc stuff that might even be moderately interesting (poor guy :depressed:, he does computer science but he tolerated it well), and sometimes he would bring up topics and then we'd discuss them. It seemed to work...I brought my score in psych/soc up from 45th percentile to 97th percentile in two months.

MATERIALS: Varied based on the section.
CHEM/PHYS: Here's where I needed the most help because I was so far out of undergrad. I purchased a Kaplan class (mainly for the online resources and not the class sections), and I read all the books, outlined them, bought a whiteboard, did problems, and watched either Khan Academy or Kaplan videos (until I realized the Kaplan class sections were kinda useless since I was online-only). I spent a ton of time on amino acids and gen chem after I realized I was never going to understand some of the physics topics (still don't get what magnetism is). Overall my biggest improvement...went from scoring about ~120ish per FL to 127 on the real thing.
CARS: I did zero studying here. Zero. I'm a speed-reader (my grandfather decided that instead of just teaching me to read he'd teach me to speed-read instead :rofl:), so I'd usually get done with CARS ~20 minutes early.
B/B: I studied my amino acids and went over my notes from grad school. Also, listened to a lot of terrible songs about various parts of metabolism (I can still sing the glycolysis song from YouTube by heart). Overall not a lot of studying here either; probably poured most of my time into the kidney since I didn't understand it well.
PSYCH/SOC: Lots of reading. Tons of reading. Kaplan mainly, and I'd outline the chapters just like for C/P, but I also watched a bunch of Khan videos on sociology since I'd never taken it before. Second biggest improvement, went from ~123/124ish to 130 on the real thing.

KAPLAN BREAKDOWNS: 1: 497, 2: 499, 3: 508, 4: 506, 5: 507, 7: 508 (may not be exact, doing from memory)
AAMC BREAKDOWNS: scored exam: 514, percentile exam was like 67th/96th/94th/77th or something.

MAJOR: Biology in undergrad, Biomedical Sciences Masters.

TIPS: RELAX. Seriously. I get that this is an important test for you. You may be freaking out about spending money that's not in your budget (I definitely postponed taking it as long as possible to keep my finances in order), but seriously, be as calm as possible, especially on test day. You're at the test center, you know all you are going to know. Now is not the time to chastise yourself for not studying fluid dynamics more. If you just go in there calm and accept that you're probably not going to know everything, you're going to be fine.
Also, I cannot stress enough the importance of taking breaks. I get that it feels like there's an insurmountable amount of knowledge for this exam, but you're not going to get anywhere if you burn yourself out to the point where you can't retain any more information. I definitely took a weeklong break about three weeks before my exam because I was hitting saturation point. I didn't want to study anymore, and I wasn't really studying effectively during that time either. So I gave myself a week off, went hiking, relaxed with my husband and friends, and got back to it the following Sunday. This is a marathon, not a sprint.
Also, take as many practice tests as possible, preferably under stressful conditions. I took all my practice tests either at my house with two rabbits running around attempting to destroy my house or at a coffeeshop sans headphones. The more distractions you expose yourself to, the more you'll look at test center conditions like heaven.

HOW LONG DID YOU STUDY?: If we count the time from buying the books to test day, about seven months. If we count from when I actually opened the books, about 2.5 months seriously. Don't rush yourself into studying if you're not ready. It's more about effective studying than just sitting down with the material.
 
Last edited:
  • Like
Reactions: 5 users
1) Your individual scores and composite score
First Attempt:
Chem/Physics: 125
CARS: 127
Bio: 127
Psych/Soc: 129
Composite: 508

Second Attempt:
Chem/Physics: 126
CARS: 132
Bio: 125
Psych/Soc: 126
Composite: 509
2) The study method used for each section
Each section was a combination of content review and practice questions and exams. It was all self-directed, no classes or private tutors. Both attempts, I gave myself about 10-12 weeks to prepare.
3) What materials you used for each section(Kaplan, TPR, Examkrackers, AAMC, TBR, etc)
I tried TPR and hated it. I felt like it was a waste of time since it covered such broad content with the hopes that it might appear on the test. It did not feel like it was relevant. For my first attempt, I used Khan Academy heavily -- watched nearly every video, especially for the science sections and did lots of practice passages. It decently prepared me, but the practice passages weren't similar enough to the real exam. The second attempt, I exclusively used Examkrackers. I loved their books and would highly recommend that you don't waste your time with TPR (I can't speak to Kaplan or other programs). EK does an excellent job of covering relevant content and providing highly realistic practice passages and exams. I had poor timing on the Chem/Physics sections on both attempts, usually rushing on the last 7 or so questions. I feel like if I were faster, my scores would have been much better, because EK did prepare me well to know the content.
Also, EK did an excellent job of preparing me for the CARS section. I was shocked when on my first attempt, I only got a 127 in CARS. On every practice exam, I had always gotten a perfect score in that section. But, I had taken it for granted and doing the occasional TPR passage didn't prepare me enough for what turned out to be a challenging CARS section on the real thing. For my second attempt, I did everything that EK told me to do, and clearly it worked.
The only thing that I feel no one does well yet is preparing students for the Psych section. The first time I took it, I felt terrible about the section and was shocked by the 129. The second time, I still felt terrible, and didn't do as well. There is still content the MCAT uses that the prep companies have not picked up on yet and it really feels like a guessing game most of the time. This is coming from someone whose major mandated psychology, communication, and sociology classes!
4) Which practice tests did you use? (Optional: include scores)
I used EK tests 1-3 (got tired of paying $50!) as well as both official AAMC tests. EK does those little half tests at the end of each chapter -- those were amazingly useful.
5) What was your undergraduate major?
Economics B.S.
6) Any other tips you may have for those of us who still have this test lurking over us?
Perhaps this is better in the "should I retake" thread, but damn, how I wish I would have just gotten my perfect CARS score the first time! I would have had a 514. Be careful and try to get it right the first time. I was so anxious about retaking because I knew that dropping a point or two was possible. Thankfully I netted one extra point, but it really could have gone the other way. Also, don't waste your time with TPR. Just use EK. If I would have used EK the first time, I think I would have gotten the score I wanted and would not have felt obligated to do the retake. Timing is crucial. Things will not go perfectly on test day (my Citrix connection was disrupted in the middle of the Chem/Physics section on my second attempt and the time continued to run even though I was locked out of the test for a few minutes). Be fast enough that losing a few minutes won't affect your ability to complete the test, like it did with me. Also, as I've been reminded by doctors and advisers and nice people, the MCAT is just ONE part of the application. I was told directly by the admissions dean for a top school that if you're over the 80th percentile, you'll be just fine. If you're on this thread shooting for the 509+, don't stress if you do "only" get a 509 or a bit lower. The average accepted score last cycle was a 506 (my premed adviser just told me that earlier today). Just do your best :)

7) How long did you study for the MCAT?
About 10-12 weeks. The first attempt, I studied over my summer break. The second attempt, I had already graduated and was working full-time. Studying was crammed to the late evenings and weekends. Not ideal (certainly doable) but if you have the luxury of a summer break, take advantage of it.
 
You could also just hit "more resources" at the top left (as opposed to "study plan") and this will bring you to a page where you can filter thru resources.

For example select "chem/phys/bio for units 3" and all the resources for those topics will show up
@Krabbeman, Im currently in the kaplan course now. You are my inspiration. You motivate me to do as much as i can in the course. I appreciate your comments. Thank you. I just have a question, how did you have time to do the recommended and extra passages? Did you complete these by the time the course was over, or did you do them after the course was over?
 
@Krabbeman, Im currently in the kaplan course now. You are my inspiration. You motivate me to do as much as i can in the course. I appreciate your comments. Thank you. I just have a question, how did you have time to do the recommended and extra passages? Did you complete these by the time the course was over, or did you do them after the course was over?
Thank you! I completed then during the course for the most part. Even tho I was in classes I pretty much totally neglected my classes and studied like 5 hours a day just for the MCAT. I hope that helps. I think if you are running out of time to do them before classes maybe you are spending a little too much time obsessing on readings. During the first content building pass of the course, your goal is just to know the material solidly but not necessarily cold.
 
  • Like
Reactions: 1 user
This might be a silly question, But does anyone think the Kaplan review videos are sufficient enough to study the material tested on the MCAT? Obviously,one will do practice questions and passages. I want to go ahead and start studying but I'm super overwhelmed by the Kaplan books 7 books in total that is an insane amount of pages to read and fully comprehend any advice on how to tackle this? is it absolutely necessary to read all of the chapters in all those review books? How many chapter does one read a day and do you focus on one subject only or do you for, example do 2 chapters of Biology and 2 of Gen Chemistry I have no idea where and how to start
 
This might be a silly question, But does anyone think the Kaplan review videos are sufficient enough to study the material tested on the MCAT? Obviously,one will do practice questions and passages. I want to go ahead and start studying but I'm super overwhelmed by the Kaplan books 7 books in total that is an insane amount of pages to read and fully comprehend any advice on how to tackle this? is it absolutely necessary to read all of the chapters in all those review books? How many chapter does one read a day and do you focus on one subject only or do you for, example do 2 chapters of Biology and 2 of Gen Chemistry I have no idea where and how to start

I am also using Kaplan's 7 books for content review. I only use the videos if I am super confused on a subject (either Kaplan/Khan/Course savers- if I can find a free video there)

This is a sample of today:
-Read Ch. 7 Gen Chem
-Read Ch. 3 of Behavioral
-AAMC Chem question pack questions #1-15 with review of content in each question if I do not already know it

then in the evening time:
-Write up/ quick review of notes for Ch 7 Gen chem in my own words
-Write up/ quick review of notes for Ch 3 Behavioral in my own words
-Anki (for stuff like A. Acids, terminology/concepts that don't stick with me, etc. At this time I also create new cards/decks for the things that aren't making sense to me after I have written up my notes for today)
-2 verbal passages from either EK 101 or the AAMC CARS question pack

When I am doing any type of practice question, I write down what I got a particular question wrong under each question. I also do this for questions that I guessed on and got right by luck. Then I throw those into a sheet that I have titled 'MISTAKES' and look for common themes on what I am missing.
It's a lot of things to know... but I think it is definitely doable.

Input to edit study schedule is also appreciated, since this is the first experience I have for studying for this beast of an exam.
 
  • Like
Reactions: 1 user
Score: 522 131/128/131/132

Method: Studied my butt off for about 8 months. As a non-traditional student, I was also taking pre-req classes during this time, which is a strategy I would highly recommend. I did a kaplan course, so I took the diagnostic, and then started into content. My advice is to go through at least half of the content before taking another practice test. If you don't learn the content, you may as well be burning the practice tests for fun. I absolutely HAMMERED the kaplan review books, except for the Ochem book. IMO, that one isn't worth it. The organic on the MCAT won't be helped by that book. Later on in my review, I focused on doing questions and reviewing them thoroughly. The AAMC materials are like gold, and should be saved for last.

Materials: Kaplan and AAMC. AAMC is definitely the best out there, so save it for the last month or two before the test.

Major: Nursing. Yes, Nursing. I basically took my prereq courses and paid very close attention to them to get the basic content down. Kaplan books helped to cover a lot I didn't know as well. I've said it before and I'll say it again.....The MCAT has more to do with how you think than with what you know. On test day, questions aren't straightforward applications of bland facts. Learn to think outside the box.

Practice Tests:
Kaplan Diagnostic: 495
Kaplan 1: 506
Kaplan 2: 506
Kaplan 3: 506
Kaplan 4: 511
Kaplan 5: 511
AAMC Unscored: 87% overall
AAMC Scored: 518

Tips: Studying for this test is a marathon, not a sprint. It is difficult to stay on-task and force yourself to study every day, but it will pay off if you do. For me, this was a career-determining test and I didn't want to leave anything up to chance. I probably could have taken it sooner and got a decent score, but I'm glad I didn't.

@RN MD 2017 Was this the first time you took the MCAT? I am also a non trad student (although I've only been out of school for 3 years, but I still don't remember anything from undergrad)
 
Thank you! I completed then during the course for the most part. Even tho I was in classes I pretty much totally neglected my classes and studied like 5 hours a day just for the MCAT. I hope that helps. I think if you are running out of time to do them before classes maybe you are spending a little too much time obsessing on readings. During the first content building pass of the course, your goal is just to know the material solidly but not necessarily cold.

Okay. Thank you. That does help!
 
1) Good morning ladies and gentleman. It is greatly appreciated to all who have shared their techniques and provided valuable feedback on their MCAT experience. I come here seeking advice on a study plan for my MCAT. First let me share a little bit about myself. I am going into my junior year in college at the age of 23. Without going into too much detail I am a paramedic in Southern California and work full time 7:00am-3:30 pm. I have been studying since the beginning of March for the MCAT exam. So far I have logged in on average approximately 3-4 hours a night after work and 7-8 hours of studying on Saturday and Sundays.

That being said, I am using TBR study materials with Gold Standard (for question), Sterling Test Prep Question Books, as my main materials. My strategy is to do 70% practice questions and based on my scores I will spend the other 30% better understanding material I may have got wrong/misunderstood. I am nervous about the Physics material mainly because I did not do so well in my college Physics courses (Physics I = B Physics II = B). I mainly attribute this to my lack of Mathematics background (I have only taken intermediate algebra in college). Does this schedule seem feasible for a January MCAT?

2)Also, When studying do you believe it is best to focus on one subject from start to finish? or would it be best to divvy up days and cover the material as a whole (for example Monday/Tuesday= BIO/Biochem, Wednesday/Thursday= Chem, Friday/Saturday= Physics, and Sunday= Ochem)

Between Now and January:
I am taking a Neurobiology course this summer and the new biochem MCAT program via Temple University in September. I have yet to sign up for courses for my Fall 2016 semester but my goal is to take courses that will complement material seen on the MCAT.
 
1)

2) The study method used for each section

C/P: Examkrackers Chem and Physics books, and many of the Khan academy passages. I think doing the Khan passages was actually pretty beneficial, I did about 70-80% of the C/P passages, and probably more than 80% of the P/S passages, but (regrettably) only a few of the B/B passages. You can see a correlation between the amount of Khan passages I did and my scores.

CARS: I read a lot for pleasure, and I also majored in Philosophy in undergrad, which helped me understand the formal and informal logic that goes into answering questions on this section. For example, I think it is important to be able to reason what MUST be true based on a passage, as well as what COULD be true, and what CANNOT be true.
Additionally, I studied for the LSAT in undergrad, which I definitely think helped prepare me for CARS. In fact, if you are having difficulty on this section and have enough lead time before the test and aren't getting results from your current prep books, it might not be a bad idea to pick up an LSAT reading comprehension book to see if you find any benefit from it. Powerscore was a good company a few years ago at least, and doing this could help with the understanding of logic I think is important to be able to distinguish between similar answers. Another option would be to pick up a book on formal and informal logic but I would try the former first.
Additionally, I think CARS is a pretty interesting section and am actually thinking about tutoring it perhaps for one of the prep companies now that I have my scores back. I'd be happy to answer any questions about it on here or in PM.

B/B: I primarily used Examkrackers books for this section. Although 127 is an OK score if I could go back and study again I would complete many of the Khan passages as well as get some of those 1001 questions in Biology and Biochem books. My biggest issue in this, C/P, and P/S were with knowledge gaps where I would encounter something I just had not seen before in my classes or prep.

P/S: I actually only took one psychology course in community college a number of years ago and didn't take sociology. I didn't take intro psych and socio courses because I figured it would be more effective to study the material expected to be on the test, and also would save several thousand in tuition and missed work.
I am guessing a number of people are also in a similar situation so what I did for this section was:

-Read an entire intro to psych textbook (a bit each week) during the months I was studying. The book I chose is Psychology: A Concise Introduction by Richard Griggs 4th edition. Personally I thought this book did a fantastic job of covering the same material that is on the MCAT. I would literally be reading one day and then would recognize something I just read a few days later while doing the Section Banks, official guide, etc. It goes into more depth than EK and probably PR, and AAMC definitely tested things (at least on practice materials) that were in this book but not in EK.
-Completed the Princeton Review Psych/Socio book and the Examkrackers book
-Completed 80%+ of the Khan passages as mentioned.


3) What materials you used for each section(Kaplan, TPR, Examkrackers, AAMC, TBR, etc): See above

4) Which practice tests did you use? (Optional: include scores)
AAMC Scored: ~511-512 with section scores: 127/~130/126/129. I took this initially in individual sections.
AAMC Unscored: 78-80% total
OG: Similar with around 85% P/S, 65% B/B and probably 75-85% C/P and I didn't do CARS on the guide
Section Banks (highly recommended like many other people suggest).
The EK books chapter and 30 min exams, and 1-2 of the free preptests from prep companies. In retrospect I wish I had bought EK and Next Step etc. full lengths but they are rather expensive at $50 bucks a pop (for EK).

5) What was your undergraduate major? Philosophy

6) Any other tips you may have for those of us who still have this test lurking over us? Start studying early depending on your level of content knowledge. The earlier you begin (even if on weekends) the more time you have to adjust to the requirements of the test.

A few other things I did which I think may have made a difference:

-Stop drinking coffee, tea, or anything with caffeine several weeks before the test. I was planning to do this anyway, but I was even more convinced when last year I had a discussion with a girl who is a student at my state's med school who scored a 39 on the prior MCAT, and she strongly suggested not drinking anything with caffeine as well. Why? Caffeine is a diuretic, and you don't want having to use the bathroom to disrupt your test day. Additionally, IMO caffeine can amplify adrenaline and anxiety, which is not what you want while testing, and can affect sleep as well. I actually weaned off all caffeine about 5-6 weeks before the test, and did not drink any for 3 weeks+ before. Don't quit caffeine the week of the test though, give yourself enough time to adapt to not having it.
I slept well the night before (see next paragraph) and was fairly calm and had minimal anxiety on test day (though more the days and weeks leading up to the test).

-Tips for getting a better night's sleep. Two nights before the test, I did not sleep almost at all. So on the day before the test I was worried I'd be up all night again, and I found this article with tips for getting to sleep: http://www.huffingtonpost.com/entry/15-ways-to-fall-asleep-faster_us_55dde3e7e4b04ae497054470

I did the socks thing, the breathing, and the shower, got some sun, and went to bed with 9 hours before I had to wake up. I woke up only once during the night, and got a good 8 hours total sleep, and woke up feeling rested.

-Eat healthy and do cardio or other exercise for endurance.

7) How long did you study for the MCAT? For more than 7-8 months total, but on weekends only as I am working as a scribe right now during the week. I think it's reasonable to start studying even earlier for this test than the old exam as there is a fair bit more material on this one and most of us have at least one or more subjects that we aren't very familiar with (whether psych, socio, etc.)
 
Last edited:
  • Like
Reactions: 1 user
1) Your individual scores and composite score
5/14/16 MCAT
517: 129, 127, 132, 129

2) The study method used for each section

3) What materials you used for each section(Kaplan, TPR, Examkrackers, AAMC, TBR, etc)

CP: Wikipremed was pretty helpful for this section, especially phys and gen chem. I supplemented with a slight bit of Khan Academy for physics.
CARS: did a few section practice tests from old Princeton Review books. I won't lie, I was pretty lazy with prepping for this section, and my score reflects this.
Bio: Kahn Academy was by far the best resource. It goes way into the weeds most of the time, but this actually helped me retain the information that I actually had to know. Listening at 1.5-2X speed helped a lot. Kimball biology pages were really good as well. The rest came from class notes from bio, micro, biochem, and cell biology
Psy/Soc: Kahn Academy was great. I honestly didn't feel like putting myself through a semester of intro soc or psych. It can all be self learned by the average person.

General prep: section banks were by far the most useful when it came to practice questions because they were harder than the real thing. I did the bio section bank twice, with once being the day before the MCAT. I also did the Khan Academy passages just to evaluate my content review. They were not the best at mimicking AAMC questions, but it was free, so no complaints from me. I also used the Kaplan 7 book set for content review. There were some spots where they emphasized low yield material and de-emphasized high yield material, but it was a decent review book if you need more extensive content review.

4) Which practice tests did you use? (Optional: include scores)
Princeton Review diagnostic. This was my first one. 506.
3 NS: best after AAMC. Matched the feel and style of the MCAT, but more difficult. It helped me to over prepare for the real thing. 508, 509, 510
2 EK: pretty decent. Don't like the fact that you get no score and the format is that of the old MCAT.
Raw scores in the mid 70s.
AAMC: I did the sample exam a week before the real thing. Raw score in the mid to upper 80s. I took the AAMC scored 4 days before the real thing. Got a 514. I felt that after taking the NS and EK exams, the AAMC practice tests were too easy, but they actually matched the difficulty of the actual exam. The only discrepancy that I came across was that the CARS section in the sample test was much easier than the one in the AAMC scored and the actual MCAT.

5) What was your undergraduate major?
Engineering

6) Any other tips you may have for those of us who still have this test lurking over us?
Like others recommended to me, don't take a full length the day before. You should probably do some light content review and some practice questions. This is what I did. Others say not to do anything except relax, but that would have actually made me less relaxed. I also for the week before tried to sleep and wake up at the same time as test day. This pretty much went to hell in a hand basket for me because I only got an hour of sleep the night before test day due to nervousness, and was pretty much a zombie that day.

7) How long did you study for the MCAT?
I started seriously studying about 6 months before. I pretty much had to relearn everything there is to know about gen chem, physics, and organic, so I had to put in an average of 3 hours a day. When I had a month to go, I ramped it up to 5 hours on weekdays.
 
1) Your individual scores and composite score

517 - 128/131/129/129

2) The study method used for each section

over christmas break I did TPR's winter bootcamp. However, they assign so much time just for review classes I did not have enough time to actually look through the review books or do any practice passages. So, about two weeks through I stopped spending all the time watching the lectures and switched to just reading the review books and doing practice passages, but I did still watch all of the CARS lectures because I thought those were helpful.

Chem/Phys - so I am a chemistry major and a physics TA so my biggest mistake was not doing enough practice passages for this section thinking that since I knew most of the material I would be fine.

CARS - I did over 50 CARS passages as well as watching all of TPR's lectures

Bio - I did a bunch of passages and read the review book

Psych/Soc - same as for Bio except I also made flashcards. However, there were a few terms on the MCAT psych/soc section that were simple definitions that I did not remember ever seeing in TPR's book so I do not know if I blanked or if they really weren't included. So to be on the safe side I would use an additional psych/soc book to supplement TPR

3) What materials you used for each section(Kaplan, TPR, Examkrackers, AAMC, TBR, etc)

TPR for everything

4) Which practice tests did you use? (Optional: include scores)

I took one TPR full length two weeks before the real thing and I got a 506 - 125/126/128/126

5) What was your undergraduate major? Chemistry

6) Any other tips you may have for those of us who still have this test lurking over us?

I think the best thing I did was that test day I brought a really good bar of chocolate and had a little of it during every break and I also brought all of my favorite foods, which I think helped keep my spirits up and kept me from getting too fatigued.

7) How long did you study for the MCAT?

4 weeks of 8-12 hours a day (over christmas break) with the exception of christmas day and the day after new years eve. (I would not recommended this though but it can be done)
 
  • Like
Reactions: 1 users
@Krabbeman
Grats on the amazing score. Were you working those 5 months?
I am contemplating on whether to return to full time work while I prep for Jan. 2017 test date.
or
Studying for MCAT and shadowing and volunteering (although I have a lot of clinical/volunteering hours already)
 
  • Like
Reactions: 1 users
@Krabbeman
Grats on the amazing score. Were you working those 5 months?
I am contemplating on whether to return to full time work while I prep for Jan. 2017 test date.
or
Studying for MCAT and shadowing and volunteering (although I have a lot of clinical/volunteering hours already)
I was taking Classes full time and working maybe... idk 20 hrs a week
 
  • Like
Reactions: 1 user
1) Scores
509, 128/129/126/126

2) Study method
CARS: Did practice passages to make sure I felt comfortable, but did not "study."
Worked my way through the Examkrackers books chapter by chapter for the rest of the material, took all 30 min chapter exams and did all in book practice problems.
After week 5, took an EK full length every weekend. Took the AAMC full length the week before my exam. When I started to burn out 2 weeks before the exam date, I supplemented with Khan Academy videos for my weakest subjects.

3) What materials you used for each section
Examkrackers 9th edition
AAMC Scored full length, section banks
Khan Academy videos
McGraw Hill 500 MCAT questions in General Chemistry (had been a long time since I took Gen Chem, wanted extra question practice)

4) Which practice tests did you use? (Optional: include scores)

EK 1-3
1- 59%
2- 70%
3- 69%

AAMC FL Scored- 506

5) What was your undergraduate major?
Business Administration

6) Any other tips you may have for those of us who still have this test lurking over us?
If I could have done it over again, I would have completed content review earlier. I didn't have a strict content review period- instead, I worked my way through the books up until 2 weeks before my exam. I wish I had more time dedicated specifically to practice questions only. Also, I started the AAMC materials too late (10 days before the test). I blame being in school/writing my thesis this semester, but I can't help but feel like I had the ability to score higher if I had devoted more time to doing practice questions.

That being said, I think that doing timed practice questions/full lengths during content review has some merit. I am so glad that I didn't wait until the very end to start taking them. Spacing them throughout your studying gives you a good feel for time management/areas of weakness.

7) How long did you study for the MCAT?
10 weeks

What was your AAMC scored FL breakdown?


Sent from my iPhone using SDN mobile
 
1) Your individual scores and composite score

517 - 128/131/129/129

2) The study method used for each section

over christmas break I did TPR's winter bootcamp. However, they assign so much time just for review classes I did not have enough time to actually look through the review books or do any practice passages. So, about two weeks through I stopped spending all the time watching the lectures and switched to just reading the review books and doing practice passages, but I did still watch all of the CARS lectures because I thought those were helpful.

Chem/Phys - so I am a chemistry major and a physics TA so my biggest mistake was not doing enough practice passages for this section thinking that since I knew most of the material I would be fine.

CARS - I did over 50 CARS passages as well as watching all of TPR's lectures

Bio - I did a bunch of passages and read the review book

Psych/Soc - same as for Bio except I also made flashcards. However, there were a few terms on the MCAT psych/soc section that were simple definitions that I did not remember ever seeing in TPR's book so I do not know if I blanked or if they really weren't included. So to be on the safe side I would use an additional psych/soc book to supplement TPR

3) What materials you used for each section(Kaplan, TPR, Examkrackers, AAMC, TBR, etc)

TPR for everything

4) Which practice tests did you use? (Optional: include scores)

I took one TPR full length two weeks before the real thing and I got a 506 - 125/126/128/126

5) What was your undergraduate major? Chemistry

6) Any other tips you may have for those of us who still have this test lurking over us?

I think the best thing I did was that test day I brought a really good bar of chocolate and had a little of it during every break and I also brought all of my favorite foods, which I think helped keep my spirits up and kept me from getting too fatigued.

7) How long did you study for the MCAT?

4 weeks of 8-12 hours a day (over christmas break) with the exception of christmas day and the day after new years eve. (I would not recommended this though but it can be done)

I love seeing stories of people who only prepped for a month or two and did great. I'll probably study a bit in the fall, but ultimately winter break will be game time for me and MCAT first date in January. Is this when you took the mcat?


Sent from my iPhone using SDN mobile
 
I love seeing stories of people who only prepped for a month or two and did great. I'll probably study a bit in the fall, but ultimately winter break will be game time for me and MCAT first date in January. Is this when you took the mcat?

Yeah! I took it on January 24th which was a saturday and it was perfect for me because I just studied a crazy amount for four weeks and then once class started in January I just did some review and more practice passages and so I didn't have to worry about getting behind in school work in order to study for the MCAT
 
1) Good morning ladies and gentleman. It is greatly appreciated to all who have shared their techniques and provided valuable feedback on their MCAT experience. I come here seeking advice on a study plan for my MCAT. First let me share a little bit about myself. I am going into my junior year in college at the age of 23. Without going into too much detail I am a paramedic in Southern California and work full time 7:00am-3:30 pm. I have been studying since the beginning of March for the MCAT exam. So far I have logged in on average approximately 3-4 hours a night after work and 7-8 hours of studying on Saturday and Sundays.

That being said, I am using TBR study materials with Gold Standard (for question), Sterling Test Prep Question Books, as my main materials. My strategy is to do 70% practice questions and based on my scores I will spend the other 30% better understanding material I may have got wrong/misunderstood. I am nervous about the Physics material mainly because I did not do so well in my college Physics courses (Physics I = B Physics II = B). I mainly attribute this to my lack of Mathematics background (I have only taken intermediate algebra in college). Does this schedule seem feasible for a January MCAT?

2)Also, When studying do you believe it is best to focus on one subject from start to finish? or would it be best to divvy up days and cover the material as a whole (for example Monday/Tuesday= BIO/Biochem, Wednesday/Thursday= Chem, Friday/Saturday= Physics, and Sunday= Ochem)

Between Now and January:
I am taking a Neurobiology course this summer and the new biochem MCAT program via Temple University in September. I have yet to sign up for courses for my Fall 2016 semester but my goal is to take courses that will complement material seen on the MCAT.

Hi there! While I can't speak to the length of your study schedule (I only did 10 weeks while I was a full time student), I can tell you not to worry about physics- I had two B's in those classes and was very apprehensive as well. There is so little physics on the new exam that I would not spend much time stressing about it. I don't remember the exact breakdown, but go look at the AAMC's guide to the new exam- it tells you the percentage of physics questions in the C/S section. If I remember correctly ~10 questions. And remember that not all of these will be straight physics questions, but may just include some basic physics concepts. Know your formulas inside and out so questions will immediately look somewhat familiar to you. I ended up getting a 128 in that section so don't worry, it definitely can be done!

Also in terms of your idea of splitting up subjects on days, I actually enjoyed alternating subjects throughout a day. So for example, I would work through a bio chapter or a set of bio passages and then move on to physics, then to CARS, and then back to bio. I think that helped me stay focused since its easy to get bored when you spend extended time on just one area!
 
1) Score: 515 - 130/128/127/130

2) Study Method: Self-Study. I spent a ridiculous amount of time not only studying but also preparing myself to study. I researched all the books I used, what schedule I'd follow, and how to study for this exam.

I graduated from college 3 years ago and hadn't had GenChem or GBio for 6. That meant I had to relearn a lot of material. Additionally, my GBio classes never covered the Endocrine, Digestive, or Nervous systems, so I was learning the material for the first time when I began to study.

If you're a nontrad, this exam is entirely doable. If you're an undergrad, find a summer or a solid chunk of time post-grad to eat, breathe, and sleep MCAT material. I'm not sure if relearning everything (or learning it for the first time in some cases) made me stronger because I had to break things down, but know that I really struggled through a lot of the material.

I do my best studying on my own, but I cannot deny how valuable and encouraging it was to be on an SDN Skype group-study. It was free, and we all encouraged one another when times got rough. Whatever you do, find a partner or (better) a set of partners to help slog through this material.

3) Materials: Even split between: KhanAcademy, EK Books, TBR Books, and TestingSolutions for CARS. Additionally, a lot of time spent on this MCAT forum and reddit.com/r/mcat.

4) Practice Tests:
Kaplan Diagnostic (day 1): 498
TPR Demo: 502
NS Diagnostic: 509
TPR 1: 502
NS 1: 510
TPR 2: 507

AAMC Sample (1 month before): 84%
AAMC FL (1 week before): 517 - 130/129/129/129

*Note. I took more than these (13 HL & FL tests). I'm just including these specifically because they show how studying increases your scores. After a month or two of content review, do 1 HL or FL test every single week, same time, different places (if possible). I took my first 8 exams in the basement of my house at 8 am every Sunday morning, then moved onto an office space.

5) Undergrad Major: Foreign Language. I completed all basic prerequisites + Genetics + Biochemistry.

6) Study Tips: Use the internet. There is so, so, so, so, so, so (so so) (so....) (s0!) much information on this website and others like it on (1) MCAT content, (2) testing strategy, and (3) how to study.

I'd also encourage people to take this test seriously to the point where you're willing to condition your body and mind to excel on the exam. For me, that meant: (1) at minimum, exercising 1 hour daily, (2) meditating and practicing mindfulness, (3) adopting a sleeping schedule conducive to the test day conditions (get your butt in bed by 11 PM at max and wake up by 7 am), (4) devouring content in as many forms as I could get it: Khan, Youtube, practice exams, practice passages (old & new), podcasts, MOOC courses (I took MIT's Intro Genetics & Bio and Yale's Psych101 course for free).

7) How long did you study?
About 3.5 months, 3 of those months full time (i.e. 7 am wake up -> 8 am library -> 4 pm home -> 5-6 pm gym, very important -> 7-10 pm Skype study group). I moved back into mom's house to be able to afford it. Giving up your independence for a few months to do well on this test is entirely worth it, if possible.



Last shout out is definitely to SDN in general. I love this website. It has helped shape who I am as a student and who I will be as a physician. Holla at me if you need help.
 
  • Like
Reactions: 3 users
1) Real MCAT 512 (131, 127, 125, 129) May 20th 2016

2) For Chem&Phys and Biochem sections I would review material by reading books or watching videos. I would many practice CARS using EK passages. For psych I would review using TPR psych book and supplement that with Khan, then practice using any material I could find.

3) I did all TPR tests. Read their CARS, psych books. The other biochem, physic books seemed too dense. My opinion is that Kaplan has the better books (excluding psych) but TPR has the better tests. EK practice tests are also very good. AAMC material are of course a must. I did the sample and scored test and every section bank they offered.

4)
TPR
Course Test 1: 496 (124, 124, 124, 124)
Course Test 2: 500(125, 124, 126, 125)
Course Test 3: 502(126, 126,125,125)
Course Test 4: 507(126,127,127,127)
Course Test 5: 501(124,124,125,128)
Complete Test 1: 503(126,125,125,127)
Complete Test 2: 502(125,123,126,127)
Demo Test: 500 (124, 126, 123, 127)
Review Test 1: 502( 125, 126, 124, 127)
Review Test 2: 499 (124, 126, 123, 126)

AAMC:
Sample Test: 78% (75%, 81%, 76%, 80%)
Guide Questions: (77%, 67%, 67%, 83%)
Biology Volume 1: 72%
Biology Volume 2: 81%
Chemistry Pack: 78%
CARS Volume 1: 73%
CARS Volume: 78%
Physics Pack: 91%
Scored Exam: 505 (127, 124, 128, 126)

ExamKrackers:
EK1: 65% (64%, 62%, 63%, 68%, 65%)
EK2: 67% (57%, 67%, 69%, 71%)

5) Bioengineering then switched to Biomedical Science

6) TIPS:

1) There will be a moment when you will break down. It is important to not let MCAT rob you of your confidence. I thought I did really well after I took the AAMC scored practice test, but then realized I got a 505. (This was a week before my April 23rd 2016 exam) I was very upset. Thankfully I had my friends and family to get through these low ends. I decided to push back to the May 20th exam. I'm glad I made that decision. Don't be afraid to push back your exam - take it when you are ready.

2) Treat each practice exam as if they were the real deal. I probably didn't need to do all the practice tests that I did. I didn't give each practice test my 100% so in the end I felt like I did more tests than I needed and felt that my level comprehension was lower than it actually was.

The tips below are geared towards CARS but good for all subjects:

1) When you are stalled on a question ask yourself "What is it that I know." Trust in what you know.

2) Breakdown the question to sub-questions. Try to figure out what these sub-questions are.

3) Choose an answer choice that is in-line with what you know. Never let an answer choice make you question what you know. In other words, try not to fall for an answer choice because it sounds good.

4) I found that my first answer choice is usually correct about 70-80% of the time. Do not change your first answer choice unless you are sure you have found additional evidence to warrant a change.

5) If you find an answer word for word in the passage - pick it. Don't overthink it. In my experience, MCAT isn't known to trick people.

6) If you have trouble deciding between two choices. Find evidence to support one or find evidence to refute the other. LOOK BACK INTO THE PASSAGE - you will always find your answer there. There are many times when just one word can tip you into choosing the correct answer choice.

7) I found that stress can make me overthink things so I try to stay calm when I'm taking the test. Relax! Think positively! When you are studying, imagine yourself walking out of the test center destroying the test. A good, positive attitude when studying AND when taking practice tests and the actual test can actually make a big difference in how you think and therefore how well you do.

8) Talk out your reasoning in your head. E.x. "This answer choice cannot be correct because...'

Biggest Tip of All:

For every practice test I did I would keep a word doc of all my mistakes. I have 40+ of these word docs each spanning the length of 20 pages (including images). The doc contains problems I missed, and vocabulary I didn't know. It also included any concepts, diagrams etc. I needed to brush up on - that were related to that particular practice test. Prep companies like TPR and Kaplan block the function to copy their questions and solutions - I think they do this to prevent people from sharing their content. But it would be great if they could add the option for users to gain quick an easy access to all the problems they missed. If you are a PC user, I used the snipping tool to override this. But be warned, I'm certain that this is frowned upon. Do not share any of their content to other people! I'll I did was saved some questions I thought would be useful in a doc and stored it only on my computer. If someone could let me know if this is okay or not please do.

In the two weeks prior to my exam, I would review all of the 40+ docs. I didn't do any practice exams, or read up on books etc. I just read those docs. That was the biggest help of all. It boosted my confidence, and I felt like I could tackle most of the questions MCAT would throw at me.

7) Every person is different but I spent 250+ hours studying for the MCAT in the span of Dec. 20th to May 19th.

Best of luck to everyone! It'll all be over soon before you know it.
 
  • Like
Reactions: 2 users
1) Real MCAT 512 (131, 127, 125, 129) May 20th 2016

2) For Chem&Phys and Biochem sections I would review material by reading books or watching videos. I would many practice CARS using EK passages. For psych I would review using TPR psych book and supplement that with Khan, then practice using any material I could find.

3) I did all TPR tests. Read their CARS, psych books. The other biochem, physic books seemed too dense. My opinion is that Kaplan has the better books (excluding psych) but TPR has the better tests. EK practice tests are also very good. AAMC material are of course a must. I did the sample and scored test and every section bank they offered.

4)
TPR
Course Test 1: 496 (124, 124, 124, 124)
Course Test 2: 500(125, 124, 126, 125)
Course Test 3: 502(126, 126,125,125)
Course Test 4: 507(126,127,127,127)
Course Test 5: 501(124,124,125,128)
Complete Test 1: 503(126,125,125,127)
Complete Test 2: 502(125,123,126,127)
Demo Test: 500 (124, 126, 123, 127)
Review Test 1: 502( 125, 126, 124, 127)
Review Test 2: 499 (124, 126, 123, 126)

AAMC:
Sample Test: 78% (75%, 81%, 76%, 80%)
Guide Questions: (77%, 67%, 67%, 83%)
Biology Volume 1: 72%
Biology Volume 2: 81%
Chemistry Pack: 78%
CARS Volume 1: 73%
CARS Volume: 78%
Physics Pack: 91%
Scored Exam: 505 (127, 124, 128, 126)

ExamKrackers:
EK1: 65% (64%, 62%, 63%, 68%, 65%)
EK2: 67% (57%, 67%, 69%, 71%)

5) Bioengineering then switched to Biomedical Science

6) TIPS:

1) There will be a moment when you will break down. It is important to not let MCAT rob you of your confidence. I thought I did really well after I took the AAMC scored practice test, but then realized I got a 505. (This was a week before my April 23rd 2016 exam) I was very upset. Thankfully I had my friends and family to get through these low ends. I decided to push back to the May 20th exam. I'm glad I made that decision. Don't be afraid to push back your exam - take it when you are ready.

2) Treat each practice exam as if they were the real deal. I probably didn't need to do all the practice tests that I did. I didn't give each practice test my 100% so in the end I felt like I did more tests than I needed and felt that my level comprehension was lower than it actually was.

The tips below are geared towards CARS but good for all subjects:

1) When you are stalled on a question ask yourself "What is it that I know." Trust in what you know.

2) Breakdown the question to sub-questions. Try to figure out what these sub-questions are.

3) Choose an answer choice that is in-line with what you know. Never let an answer choice make you question what you know. In other words, try not to fall for an answer choice because it sounds good.

4) I found that my first answer choice is usually correct about 70-80% of the time. Do not change your first answer choice unless you are sure you have found additional evidence to warrant a change.

5) If you find an answer word for word in the passage - pick it. Don't overthink it. In my experience, MCAT isn't known to trick people.

6) If you have trouble deciding between two choices. Find evidence to support one or find evidence to refute the other. LOOK BACK INTO THE PASSAGE - you will always find your answer there. There are many times when just one word can tip you into choosing the correct answer choice.

7) I found that stress can make me overthink things so I try to stay calm when I'm taking the test. Relax! Think positively! When you are studying, imagine yourself walking out of the test center destroying the test. A good, positive attitude when studying AND when taking practice tests and the actual test can actually make a big difference in how you think and therefore how well you do.

8) Talk out your reasoning in your head. E.x. "This answer choice cannot be correct because...'

Biggest Tip of All:

For every practice test I did I would keep a word doc of all my mistakes. I have 40+ of these word docs each spanning the length of 20 pages (including images). The doc contains problems I missed, and vocabulary I didn't know. It also included any concepts, diagrams etc. I needed to brush up on - that were related to that particular practice test. Prep companies like TPR and Kaplan block the function to copy their questions and solutions - I think they do this to prevent people from sharing their content. But it would be great if they could add the option for users to gain quick an easy access to all the problems they missed. If you are a PC user, I used the snipping tool to override this. But be warned, I'm certain that this is frowned upon. Do not share any of their content to other people! I'll I did was saved some questions I thought would be useful in a doc and stored it only on my computer. If someone could let me know if this is okay or not please do.

In the two weeks prior to my exam, I would review all of the 40+ docs. I didn't do any practice exams, or read up on books etc. I just read those docs. That was the biggest help of all. It boosted my confidence, and I felt like I could tackle most of the questions MCAT would throw at me.

7) Every person is different but I spent 250+ hours studying for the MCAT in the span of Dec. 20th to May 19th.

Best of luck to everyone! It'll all be over soon before you know it.
Thank you so much for such a great feedback and congrats on your amazing score. My question is, what did you do between April 23rd and May 20? did you do more practice tests to boost your confidence? and how often? Thanks :)
 
1) Real MCAT 512 (131, 127, 125, 129) May 20th 2016

2) For Chem&Phys and Biochem sections I would review material by reading books or watching videos. I would many practice CARS using EK passages. For psych I would review using TPR psych book and supplement that with Khan, then practice using any material I could find.

3) I did all TPR tests. Read their CARS, psych books. The other biochem, physic books seemed too dense. My opinion is that Kaplan has the better books (excluding psych) but TPR has the better tests. EK practice tests are also very good. AAMC material are of course a must. I did the sample and scored test and every section bank they offered.

4)
TPR
Course Test 1: 496 (124, 124, 124, 124)
Course Test 2: 500(125, 124, 126, 125)
Course Test 3: 502(126, 126,125,125)
Course Test 4: 507(126,127,127,127)
Course Test 5: 501(124,124,125,128)
Complete Test 1: 503(126,125,125,127)
Complete Test 2: 502(125,123,126,127)
Demo Test: 500 (124, 126, 123, 127)
Review Test 1: 502( 125, 126, 124, 127)
Review Test 2: 499 (124, 126, 123, 126)

AAMC:
Sample Test: 78% (75%, 81%, 76%, 80%)
Guide Questions: (77%, 67%, 67%, 83%)
Biology Volume 1: 72%
Biology Volume 2: 81%
Chemistry Pack: 78%
CARS Volume 1: 73%
CARS Volume: 78%
Physics Pack: 91%
Scored Exam: 505 (127, 124, 128, 126)

ExamKrackers:
EK1: 65% (64%, 62%, 63%, 68%, 65%)
EK2: 67% (57%, 67%, 69%, 71%)

5) Bioengineering then switched to Biomedical Science

6) TIPS:

1) There will be a moment when you will break down. It is important to not let MCAT rob you of your confidence. I thought I did really well after I took the AAMC scored practice test, but then realized I got a 505. (This was a week before my April 23rd 2016 exam) I was very upset. Thankfully I had my friends and family to get through these low ends. I decided to push back to the May 20th exam. I'm glad I made that decision. Don't be afraid to push back your exam - take it when you are ready.

2) Treat each practice exam as if they were the real deal. I probably didn't need to do all the practice tests that I did. I didn't give each practice test my 100% so in the end I felt like I did more tests than I needed and felt that my level comprehension was lower than it actually was.

The tips below are geared towards CARS but good for all subjects:

1) When you are stalled on a question ask yourself "What is it that I know." Trust in what you know.

2) Breakdown the question to sub-questions. Try to figure out what these sub-questions are.

3) Choose an answer choice that is in-line with what you know. Never let an answer choice make you question what you know. In other words, try not to fall for an answer choice because it sounds good.

4) I found that my first answer choice is usually correct about 70-80% of the time. Do not change your first answer choice unless you are sure you have found additional evidence to warrant a change.

5) If you find an answer word for word in the passage - pick it. Don't overthink it. In my experience, MCAT isn't known to trick people.

6) If you have trouble deciding between two choices. Find evidence to support one or find evidence to refute the other. LOOK BACK INTO THE PASSAGE - you will always find your answer there. There are many times when just one word can tip you into choosing the correct answer choice.

7) I found that stress can make me overthink things so I try to stay calm when I'm taking the test. Relax! Think positively! When you are studying, imagine yourself walking out of the test center destroying the test. A good, positive attitude when studying AND when taking practice tests and the actual test can actually make a big difference in how you think and therefore how well you do.

8) Talk out your reasoning in your head. E.x. "This answer choice cannot be correct because...'

Biggest Tip of All:

For every practice test I did I would keep a word doc of all my mistakes. I have 40+ of these word docs each spanning the length of 20 pages (including images). The doc contains problems I missed, and vocabulary I didn't know. It also included any concepts, diagrams etc. I needed to brush up on - that were related to that particular practice test. Prep companies like TPR and Kaplan block the function to copy their questions and solutions - I think they do this to prevent people from sharing their content. But it would be great if they could add the option for users to gain quick an easy access to all the problems they missed. If you are a PC user, I used the snipping tool to override this. But be warned, I'm certain that this is frowned upon. Do not share any of their content to other people! I'll I did was saved some questions I thought would be useful in a doc and stored it only on my computer. If someone could let me know if this is okay or not please do.

In the two weeks prior to my exam, I would review all of the 40+ docs. I didn't do any practice exams, or read up on books etc. I just read those docs. That was the biggest help of all. It boosted my confidence, and I felt like I could tackle most of the questions MCAT would throw at me.

7) Every person is different but I spent 250+ hours studying for the MCAT in the span of Dec. 20th to May 19th.

Best of luck to everyone! It'll all be over soon before you know it.
I got a 506 3 weeks before my test date. How did you feel on the real exam compared to the scored FL 1? My practice AAMC scored breakdown was: 127/126/128/125. I could tell how each section felt at the end of every section but on the real deal, I couldn't tell as well


Sent from my iPhone using SDN mobile
 
I got a 506 3 weeks before my test date. How did you feel on the real exam compared to the scored FL 1? My practice AAMC scored breakdown was: 127/126/128/125. I could tell how each section felt at the end of every section but on the real deal, I couldn't tell as well


Sent from my iPhone using SDN mobile


From looking at other posts, it seems like the scored full length practice test is a reliable indicator for the real thing. Usually most people get at least the score they got on practice test. For me, I obviously got a big jump. But I did take the scored practice test a month before the real thing and definitely under poorer conditions. Looking back at my mistakes for the test, I probably should have gotten a 508 or 509.

I felt like I did pretty good on the practice test but I ended up getting a 505. And on the real deal, I felt like I did pretty well on sections 1,3, and 4 but bombed the CARS section. Obviously that didn't happen so it seems like my radar is all over the place. I did however feel like I'd get around 513 after I took my test and low and behold I got a 512.

So I don't know what you can take from all this.
 
From looking at other posts, it seems like the scored full length practice test is a reliable indicator for the real thing. Usually most people get at least the score they got on practice test. For me, I obviously got a big jump. But I did take the scored practice test a month before the real thing and definitely under poorer conditions. Looking back at my mistakes for the test, I probably should have gotten a 508 or 509.

I felt like I did pretty good on the practice test but I ended up getting a 505. And on the real deal, I felt like I did pretty well on sections 1,3, and 4 but bombed the CARS section. Obviously that didn't happen so it seems like my radar is all over the place. I did however feel like I'd get around 513 after I took my test and low and behold I got a 512.

So I don't know what you can take from all this.
I mean you did well on the practice test but scored lowed in CARS and Psych. I had the exact same C/P and B/B section scores as you (127 and 128). Psych on the practice FL was a section I rushed through and lo and behold it was my lowest (125).

You said you felt like you'd get a 512-513 post-test. Is there a particular feeling attached to that estimation? haha, the reason I ask is because I didn't have any particular feeling during the B/B and Psyc parts but I wasn't feeling like I bombed any part of the test
 
1) Score: 510 Physics: 128 CARS: 126 Bio: 128 Psych: 128

2) Study Method: I studied for about 2.5 months using Kaplan and occasional Khan academy videos.

3) Study Material: Kaplan for every section, Khan academy for P/S, and TPR Hyperlearning for CARS.

4) Practice Tests:
Kaplan 1: 503
Kaplan 2: 507
Kaplan 3: 506
NS1: 510
NS2: 510
NS3: 511
NS4: 508
AAMC Unscored: 86.5% P/C: 85% CARS: 83% B/B 86% P/S 92%
AAMC Scored: 514 P/C: 130 CARS: 128 B/B: 128 P/S: 128

5) Undergrad Major: Biochemistry and Molecular Biology.

6) Tips: Do not let yourself get stressed about this test. The night before mine I was unable to sleep because I was so anxious and could barely pay attention during the actual thing. If I could go back and retake I would've spent more time on CARS doing as much practice as possible. I recommend using Anki for P/S and Khan academy videos were pretty good as well. Overall, I think it's 10x more important to do practice problems than to study information over and over again, which was probably where I went wrong. Get your hands on as much practice material as possible, and make sure to especially review what you are missing. Although I'm not completely disappointed with my score, I think I could've done better had I focused more on doing practice problems.

7) How long did you study for the MCAT? 2.5 months about 6 hours per day.
 
1) Overall Score: 520 (98th percentile) PS: 131 CARS: 130 BS: 131 Psych: 128

2) Study Method: Physical Sciences has always been my weakest subject area, so I used both Kaplan and TPR books for these to make sure I fully covered the material since I had a poor foundation. I read each of the books a few times making notes as I went along and mostly focused on concepts, relationships between variables, and memorizing formulas for this section.
CARS I didn't study for other than the practice tests since I thought my time was better spent learning science content.
Biological sciences I spent most of my study time on; I read the Kaplan bio book multiple times until I felt comfortable explaining each topic, and also read through my notes from the biochemistry course I took the semester leading up to the test- I don't think I would have succeeded on this section without having taken that course, but I also did not have a science background so this may not apply to everyone. I memorized all the amino acids, hormones, enzymes, etc using flashcards and writing out different pathways multiple times.
Behavioral sciences I used TPR + Kaplan + glossary for intro to sociology text + Khan Academy videos to cover all my bases with it being a new section. I wrote out flashcards to memorize all of the terms in Kaplan. If I were taking the test again, I would probably have taken an intro psych/soc course or at least read a textbook as there were topics on my test that I had not seen in any of my prep materials.

3) Study Materials: Kaplan 7 book 2015 set**, Kaplan 528 book (didn't find this very helpful, it seemed mostly strategy based), TPR Chem/Phys, TPR Behavioral Sciences, Lehninger Biochemistry, intro to sociology textbook glossary, Kaplan online practice tests, AAMC FL, AAMC questions packs, AAMC Official Guide Questions, Khan Academy videos (mostly psychology and some biology)
** I didn't use the CARS book at all, and the orgo book I didn't spend much time on either. The other subjects I read through multiple times each until I was confident about the material and did all the practice questions. For physics, I picked out the formulas I thought would be most biologically relevant and memorized a sheet of them for the test; I found this more helpful than doing the practice problems in the Kaplan physics text.

4) Practice Tests: I used the online Kaplan FLs that came with the course, along with their initial half-length diagnostic. Also took a TPR FL, AAMC FL, and the AAMC Official Guide as a half length.


Kaplan FL Scores: (not in order)
Diagnostic: 504
FL 3: PS:125 CARS:128 BS:125 Psych:127 Total: 505
FL 4: PS:125 CARS:127 BS:125 Psych:127 Total: 504
FL 5: PS:124 CARS:126 BS:125 Psych:127 Total: 502
FL 8: PS:125 CARS:128 BS:126 Psych:124 Total: 503
FL 9: PS:125 CARS:128 BS:126 Psych:124 Total: 503

AAMC FL: PS: 78% CARS: 91% BS: 88% Psych: 83%

AAMC Official Guide: PS: 80% CARS: 90% BS: 76% Psych: 90%

5) Undergraduate major: Marketing, but I completed a post-bac program this year which included all pre-reqs

6) Tips: Can't speak for every test day, but my exam had a heavy biochem emphasis on both science sections, so I would recommend spending a larger amount of time studying those concepts than say orgo reactions (definitely memorize amino acids!). Physics was supposed to be de-emphasized vs. the old test, but I still had a number of physics questions, so I wouldn't blow off studying that either. Overall I think the Kaplan books did a great job of covering pretty much everything with the exception of behavioral sciences, so I think they are a pretty safe bet.

7) Time Spent Studying: I completed a 1-year post bac premed program, so I had taken all of the prerequisite coursework + biochem in the year leading up to the test (May 2015). I also was enrolled in the Kaplan on site course this spring, but only really used their book set for content & FLs as I didn't find their strategies helpful. I started studying MCAT specific materials around January, took my first diagnostic in February, and took a practice test roughly once per week starting around March. In the last two weeks before the test I stopped taking practice tests and spent about 8-10 hrs a day studying content, and took the AAMC FL roughly 1 week prior to the test date.

Could you tell more how did you prepare for physics? Because, I found that Kaplan Physics book does not help a lot.
 
1) 521 total 130 PC/129 CARS/130 Bio/132 Psy/Soc
2)Explicitly used Kaplan's materials and study guide for each section
3)I used only Kaplan and aamc material. Kaplan for content studying. Kaplan Q-bank and aamc Q packs for daily practice. Kaplan FLs 1-7. And AAMC FL right before exam. Also did the 120 Qs in the official guide to the MCAT.
4)Kaplan FLs 1-7 scoring ranged from 501-508. AAMC practice right before exam ~86%
5)Biology @ top 100 state school
6)I did every single little thing my Kaplan Course had to offer. I swear by them, they helped me so much. Which is why I now teach for them:) I strongly suggest taking a course. When you do, do every single reading, watch every single video, every practice Q they got. Suck everything you can out of that course.

My schedule was this. I followed their instructions to a Tee, except didn't just do "recommended" material, I did everything the course had to offer. around 5 hours a day. When the course was over, I still had 8 weeks till my exam. So I took a FL every weekend, and spent the rest of the week doing focused content on what I got wrong. And doing practice specific to that area. Thats it folks. I simply followed Kaplan, and did FLs after the course ended.

One thing I think REALLY heped.... do one practice passage from each of the 4 section EVERY DAY (except test days). I used the aamc Qpacks and official guide, and the Kaplan Q bank for these practice passages. Don't skip this ever. Do them. Do them Do them Do them. Oh one more thing. DO THEM. one P/C, one CARS, one BIO, and one Psy/soc every day.

5) 5 months for at least 5 hours a day...... yes folks... thats 5 x 30 x 5=750 hours... its actually more than that because FLs were 7 hours long.

Note: I'm not that bright... AT ALL. I do have a 3.99gpa, but thats only because I work myself to the bone!!!! Most people won't have to put in nearly 750 hours to do this well... probably around 300-350 from what I can see. BUT... if you're dumb like me... DON"T BE DISCOURAGED!!!! JUST WORK YOUR @$$ off. You can do it! WHO CARES IF THEY SAY 300-350 hours to prepare???!!! TRY TO DOUBLE THAT IF YOU KNOW YOUR NOT SMART!! I guarantee you can out work people smarter than you, and grab a disgusting score.

Good luck


Hello! I am enrolled in Kaplan online course. And they offer AAMC question packs. But I have found that they are timed. How did you do one AAMC passage each day? I just did not see any option for that. I do not want to do all 120 questions in 3 hours. Could you help me with that? Thank you!
 
Hello! I am enrolled in Kaplan online course. And they offer AAMC question packs. But I have found that they are timed. How did you do one AAMC passage each day? I just did not see any option for that. I do not want to do all 120 questions in 3 hours. Could you help me with that? Thank you!

Hey! I am in an Kaplan course too, and so I know exactly what you are talking about when it says the AAMC materials are "timed". I think that is just a suggested time, because when you actually go and open the packs they are not timed. An interesting note though, when on the AAMC site from the Kaplan AAMC materials link once you've done the problems, it says that the number of starts is unlimited... while I've heard that it should only be 20 starts?

But to answer your question again it's not actually timed and you can pause it anytime and return.

Source: personal experience
 
Hey! I am in an Kaplan course too, and so I know exactly what you are talking about when it says the AAMC materials are "timed". I think that is just a suggested time, because when you actually go and open the packs they are not timed. An interesting note though, when on the AAMC site from the Kaplan AAMC materials link once you've done the problems, it says that the number of starts is unlimited... while I've heard that it should only be 20 starts?

But to answer your question again it's not actually timed and you can pause it anytime and return.

Source: personal experience
Thank you very much!

About 20 starts... I have found info on SDN forums about that. People say that actually if you are enrolled in Kaplan course, AAMC starts are unlimited until your course expires. By the way, when are you taking MCAT?)
 
Hello! I am enrolled in Kaplan online course. And they offer AAMC question packs. But I have found that they are timed. How did you do one AAMC passage each day? I just did not see any option for that. I do not want to do all 120 questions in 3 hours. Could you help me with that? Thank you!
Hey! When I took the course you could answer and review questions right after- and then come back to them any time you want! I'm not sure if that has changed, but I don't think it has
 
1) Your individual scores and composite score
Total: 512 PS: 127 CARS: 128 BIO:129 PSY: 128
2) The study method used for each section

This time I used Nymeria's MCAT Plan adapted from SN2ed which consists of one chapter of info and practice of each subject (Chem, Bio, Ochem, etc) a week and intense practice. I also took Florida Atlantic University's MCAT prep course but did not find it very useful.
3) What materials you used for each section(Kaplan, TPR, Examkrackers, AAMC, TBR, etc)
I used mostly Berkely Review for Chemistry, Physics and OChem and supplemented with Princeton Review. For Biology I used ExamKrackers. For CARs I used a combination of Princeton, NextStep, and Examkrackers. For psychology I used Princeton Review.
4) Which practice tests did you use? (Optional: include scores)
I took Princeton Review practice tests, Examkrackers CBT 1 and 4, Next Step Test 1,3, and 4, AAMC Sample test and AAMC Official Practice Test (I got a 506).
You can see a breakdown of my practice exam scores here.
5) What was your undergraduate major?
Biological Science
6) Any other tips you may have for those of us who still have this test lurking over us?
I rushed to take the old MCAT without really studying before it changed and got 22/32nd percentile so I can not stress enough to practice! Use whatever material you can get your hands on, from EK 1001 to the full lengths available. Take your time to understand the information, make connections between subjects and see the overall picture.
7) How long did you study for the MCAT?
3 months
 
Last edited:
  • Like
Reactions: 3 users
1) Overall Score: 522 (99th percentile) PS: 131 CARS: 129 BS: 132 Psych: 130


2) Study Method: I began studying at the end of September 2015 and took the exam June 2, 2016, which gave me about 8 months to study. I spent a few hours studying during the Fall and Winter semesters and also a few hours a day during the Christmas break. I spent the majority of this time studying the Kaplan Books.

Beginning in December, I started doing 10 AAMC practice problems a day and continued doing this until I took the actual exam. This was definitely one of the most helpful habits. It helped me see my weaknesses and strengths throughout the entire study process.

When summer 2016 came, I had 7.5 weeks until the exam. During this time, I studied 60 hours a week (10 hours a day except for Sundays). I called this my “MCAT Bootcamp.” :p At this point I began doing more practice tests, practice problems, and began reviewing the Khan Academy materials.

As far as practice tests go, I found them to not be very effective except for the 2 official AAMC exams.Besides that, I stuck to the free materials (3 from Kaplan, 1 from Princeton Review, 1 from First Step).

The practice problems on Khan Academy are not very good, especially the passage based problems. They are too detailed. I mostly just did the practice problems they had between chapters, which were decent. I would recommend sticking to the practice problems from the AAMC. I did all of them, including the flash card pack.

One of my biggest recommendations is to use notecards. A lot of people I know really like using electronic notecards via Anki, which I think is good. I personally prefer to use paper note cards because I can draw things easier and faster. Throughout the entire study process, I made note cards for anything that was new to me or that I thought would be important to remember right before the exam. Then during MCAT Bootcamp, I reviewed every note card I had ever made since I began my MCAT preparation. And during the final 2 days before the exam I went over note cards that I had gotten wrong or found particularly important one last time. This way I knew I wasn’t missing any pertinent information going into test day.

Also, about a week before the exam, I made note cards for every equation out of the Kaplan MCAT Book that I felt was useful (you gain a concept of what equations are useful through doing AAMC practice materials). Then I reviewed this stack a few times before taking the real exam. I wouldn’t recommend spending an excessive amount of time memorizing the equations, but rather gaining an intuitive understanding of the concepts behind each equation. I wouldn’t spend more than a few hours committing equations to memory.

As far as the Critical Reading section goes, I just made sure to read the newspaper everyday and do a good amount of practice problems. I would stick with the practice problems from the AAMC (and maybe Kaplan). You want to gain a gut feeling for what the AAMC testmakers are looking for.

If any of you are reading this and still have a lot of time to start your MCAT preparation, I would recommend to start now. Ideally, I would have started during my first semester of college by reading the Kaplan books while taking the related class. For example. I took Physiology the semester before I took the MCAT. I used the Kaplan MCAT Physiology book as my supplemental textbook. It helped me pay closer attention to MCAT information when my teacher talked about it during class. It helped me do well in the class (got an A!) and do well on that MCAT section (my best one at a 132!). But, no matter how much time you have, I would definitely suggest starting now.

I am glad I didn’t waste my money to do a MCAT prep program that forces you to study a set amount each week. During the semester I only had a few hours a day to study. Most of my friends were being forced to study at the pace of Altius (20 hours a week) while trying to maintain grades in their classes. If you need to have someone pushing you, I would suggest simply hiring a personal tutor, maybe even a friend you know that did well on the exam. You could just meet with them once a week to go over questions or concepts you are having a hard time with. Make a plan with them and stick to it the best you can. No need to spend thousands of dollars on a prep course in my opinion. There are a lot of online tutors out there that charge a lot less than mainstream prep companies.

3) Study Materials:

In order of usefulness:

  1. AAMC Practice Materials - I can’t stress this enough. These guys are the people who wrote the real exam. Other practice materials will require you to stress out about unimportant information.

  2. Kaplan 7-Book Series and Practice Exams - Every book in this series was awesome. Thankfully the Organic Chemistry book only takes about a day or two to read fully. I wouldn’t waste your time reading the strategies in the Critical Reading Book. I felt like any strategy beyond reading and answering the questions just overcomplicates this section. Kaplan takes it beyond over complication. I just used the practice questions from this book.

  3. Khan Academy Videos and Practice questions between chapters - The Khan Academy materials were created in partnership with the AAMC, so they are a definite must. I found it extremely helpful to watch these after having read the Kaplan books. The AAMC doesn’t clearly define what terms they are going to use for things and what aspects of topics they are going to test you on, so it is helpful to hear things from multiple perspectives. This is especially important for the Behavioral Sciences section. By the time I took the MCAT, I had heard almost every concept explained from a professor, a school textbook, the Kaplan books, and finally the Khan Academy videos. I watched every Khan Academy Video at 2x the speed and it went pretty fast. I didn’t watch the chemistry and physics videos because I felt like Khan Academy went into to much depth and that Kaplan had covered it sufficiently for me. Also, Sal Khan teaches many of these videos and he talks a little too slow for me. :p

4) Practice Tests:

Kaplan FL Scores: (not in order)

9/24/2015 - AAMC Official Practice Exam 1: 76.13%

1/2/2016 - AAMC Official Practice Exam 2: 81.16%

4/21/2016 - Kaplan Full Length 1: 80.45%

4/25/2106 - Kaplan Full Length 2: 81.82%

4/28/2016 - Kaplan Full Length 3: 83.61%

5/2/2016 - Next Step Full Length 1: 77.25%

5/5/2016 - The Princeton Review Full Length 1: 74.70% (at this point I realized that my time would be better spent reviewing concepts, so I stopped doing practice tests and spent my time watching Khan Academy videos instead. I am so glad I did. Non-AAMC Practice Tests mainly just make you feel terrible).

5/23/2016 - Official AAMC Practice Exam 1 (second time): 95.57%

5/31/2016 - Official AAMC Practice Exam (Scored) 1 (second time): 525


5) Undergraduate major: Information Systems. I don’t think it matters what your undergraduate major is as long as you take all the medical school pre-requisites before you take the MCAT, including the ones that some people say don’t help much with the MCAT (Genetics, Sociology, etc.). The MCAT is testing your ability to speak the language of science. The more pre-requisites you get under your belt, the more fluent you become.


6) Tips: Make sure to continue doing those things that keep you balanced. For me, that included spending time with my wife and family. My wife had a baby one month before I took the MCAT, so I made sure to stop studying by 6 PM every night so that I could spend time with her and our baby. I also made sure to continue exercising and eating healthy. Most importantly, I continued to pray, read the scriptures, and attend church. I couldn’t have done as well as I did without God’s help. Whatever your religious faith is, make sure to live it to the fullest even when you are busy studying for the MCAT.

Also, I felt super stressed out on test day. Every question that I wasn’t sure about added to my stress. After the Chemical and Physical Sciences section I wanted to just give up, go home, and take it two weeks later. I thought I had failed that section horribly. I guessed my way through an entire organic chemistry passage and couldn’t figure out the answers for several other problems in that section. I prayed for help and decided to continue the exam. The next few sections felt much better, but I was still seriously considering cancelling my scores and taking it again in two weeks. But, once I finished the test, I prayed and felt peace about just accepting the score from this exam. I literally cried on the way home from taking the exam.

This doesn’t have to be your test day experience. I think it would have helped me to realize that the difficulty of the MCAT exam you get doesn’t matter. It is all scored on a scale in the end anyways. And, they don’t even tell you what percent you got wrong or right in your score report. Also, they throw out questions that arn’t worded well or that too many people miss. The section I thought I had failed ended up being my second best section (a 131!), and the section that I felt the best about, Critical Reading, ended up being my worst section (a 129). So, on test day, just relax and remember that everyone is taking the same test and all that matters is that you do the best you can do and leave the rest in God’s hands. It will all work out.

7) Time Spent Studying: I have no idea how many hours I spent studying specifically for the MCAT, but it was a lot. But, it never felt overwhelming or like I didn’t have a life while I was studying for it. If you just do a little bit everyday over a long period of time, you will do great.
 
Last edited:
  • Like
Reactions: 21 users
Top